NBME 16 help

This forum made possible through the generous support of SDN members, donors, and sponsors. Thank you.

shubz123

New Member
10+ Year Member
Joined
Sep 10, 2012
Messages
1
Reaction score
0
Hi all,

Members don't see this ad.
 
Last edited by a moderator:
ok i can see your thought process now, but the big picture point is that to do any kind of cath, you have to go in from the aorta, so your first stop on the train is going to be either the LCA or RCA. for the purposes of step 1, the circ is a terminal branch, so you cant climb up there to get to the LAD
 
A 23-year-old man comes to the Emergency department because of a 2-week history of back pain refractory to treatment with over-the-counter medications. He demands a prescription for an opioid drig because ''they are the only medications that decrease my pain''. He becomes upset when is told that over-the-counter medications should relieve his symptoms. His temperature is 37.1C, pulse is 70/min, respirations are 18/min, and blood pressure is 120/70mmHg. Physical examination shows no abnormalities. Which of the following is the most appropriate action by the physician?

A) Determine which drugs have been prescribed for the patient in the past
B) Obtain serum toxicology screening on the patient
C) Order an MRI of the spine
D) Prescribe only a 2-week course of a narcotic medication for the patient
E) Refer the patient to a drug addiction program


I put B and I got it wrong. I put B because I think he is a drug abuser, isn't it?? He wants drug and becomes upset!

What is the right answer?
 
Members don't see this ad :)
on the 5% chance you aren't just trolling me, A is the correct answer.


for these "soft-skills" questions the answer is always whatever is non-invasive, non-judgemental, etc. a good way to think about them is preface the questions with "an angry black man on PCP walks into your office..." you want to be super friendly and accommodating.
 
  • Like
Reactions: 1 users
Thank you man, I'm not trolling, I thank you all for the answers, unfortunately NBME doesn't give the right answers
 
no i understand, its a pain. have you done usmleworld, by chance? it is an excellent resource for learning some of the tricks you seem to be having trouble with.
 
I did uwolrd for step1 almost twice! I study every day, but i took only 370 (=200) in NBME yesterday! I think to pass i need 192, but 200 is low result!
 
43. During an experiment, the concentration of glycolytic intermediates is measured in human endothelial cells incubated with
30 mM(54O mg/dL) glucose compared with 5 mM (90 mg!dL) glucose. Results show:
30 mM Glucose
Intermediate (Compared With 5 mM Glucose)
Glucose 6-phosphate 180%
Fructose 1,6-bisphosphate 220%
Glyceraldehyde 3-phosphate 210%
1 ,3-Bisphosphoglycerate 75%
Pyruvate 70%
Based on these data, the activity of which of the following enzymes is most likely inhibited indirectly by increased glucose
concentrations’?
A) Aldolase
B) Glyceraldehyde-3-phosphate dehydrogenase
~ C) Hexokinase
D) Phosphofructokinase
E) Phosphoglycerate mutase
F) Pyruvate kinase

Is B the right one? Thank you~
 
Yes, B) Glyceraldehyde-3-phosphate dehydrogenase is the right one, because 1,3-bisphosphoglycerate and pyruvate are the only ones <100% and are both downstream the enzyme Glyceraldehyde-3-phosphate dehydrogenase (in glycolysis pathway)
 
Last edited:
  • Like
Reactions: 1 user
A 27-year-old nulligravid woman comes to the physician because of the inability to concieve for 2 years. Menses have occurred at 3-4 month intervals since menarche at the age of 14 years. Physical and pelvic examinations show no abnormalities. Her serum FSH, LH and estrogen concentrations are within the reference ranges. Which of the following is the most appropriate pharmacotherapy for this patient?

A) Clomiphene
B) Low dose estrogen
C) Medroxyprogesterone
D) Metformin
E) Tamoxifen

I put B and it is wrong. What is the right answer?

What is the diagnosis? It's not PCOS because LH and other hormones are in range. So??
 
clomiphene. i only got it right because that's what it says in first aid rapid review
 
  • Like
Reactions: 1 user
A 65-year-old woman comes to the physician because of a 3month history of headache, weakness of her arms, and left flank pain; she also has a 14-kg weight loss during this period. Physical examination shows weakness of the proximal upper and lower extremity muscles. There is augmentation of strength with repetitive testing of the deltoid muscles. An MRI of the brain shows a single well-demarcated mass surrounded by edema in the right frontal lobe. A stereotactic biopsy specimen of the lesion shows a malignant, small blue cell neoplasm that expresses citokeratin, chromogranin, and synaptophysin. Which of the following is the most likely diagnosis?

A) Anaplastic ependymoma
B) Extranodal primary central nervous system lymphoma
C) Glioblastoma multiforme
D) Primary cerebral neuroblastoma
E) Pulmonary small cell carcinoma metastatic to the brain

I put D and I got it wrong. What is the right answer?

I was distracted by the flank pain (->neuroblastoma)
 
A 65-year-old woman comes to the physician because of a 3month history of headache, weakness of her arms, and left flank pain; she also has a 14-kg weight loss during this period. Physical examination shows weakness of the proximal upper and lower extremity muscles. There is augmentation of strength with repetitive testing of the deltoid muscles. An MRI of the brain shows a single well-demarcated mass surrounded by edema in the right frontal lobe. A stereotactic biopsy specimen of the lesion shows a malignant, small blue cell neoplasm that expresses citokeratin, chromogranin, and synaptophysin. Which of the following is the most likely diagnosis?

A) Anaplastic ependymoma
B) Extranodal primary central nervous system lymphoma
C) Glioblastoma multiforme
D) Primary cerebral neuroblastoma
E) Pulmonary small cell carcinoma metastatic to the brain

I put D and I got it wrong. What is the right answer?

I was distracted by the flank pain (->neuroblastoma)

50% of adult brain cancers are mets, and the augmentation of muscle strength with repeated testing suggests lambert eaton, so small cell lung cancer metastasis is the answer.
 
Members don't see this ad :)
A 9-month old boy is brought to the Emergency department by his mother because of severe respiratory distress for 1 hour. His respirations are 80/min. He is at 3rd percentile for lenght and weight. Physical examination shows cyanosis. Despite appropriate measures, the patient dies. Autopsy findings show a ventricular septal defect and persistent truncus arteriosus. Which of the following populations of cells was most likely absent during early fetal cardiac development in this patient?

A) Ectodermal neural crest
B) Extraembryonic mesoderm
C) Mantle layer neuroblasts
D) Mesothelial cells
E) Spalnchnopleuric mesoderm

I put D and I got wrong. What is the right answer?
 
A 9-month old boy is brought to the Emergency department by his mother because of severe respiratory distress for 1 hour. His respirations are 80/min. He is at 3rd percentile for lenght and weight. Physical examination shows cyanosis. Despite appropriate measures, the patient dies. Autopsy findings show a ventricular septal defect and persistent truncus arteriosus. Which of the following populations of cells was most likely absent during early fetal cardiac development in this patient?

A) Ectodermal neural crest
B) Extraembryonic mesoderm
C) Mantle layer neuroblasts
D) Mesothelial cells
E) Spalnchnopleuric mesoderm

I put D and I got wrong. What is the right answer?

Neural crest cells form the aorticopulmonary septum and endocardial cushions.
 
  • Like
Reactions: 1 user
Two days after undergoing a right hip replacement operation, a 67-years-old woman has shortness of breath and pain on the side of her back with deep breathing. She appears anxious. Her pulse is 100/min, and respirations are 24/min. A pleureal friction rub is heard in the right lung base. Examination of the right calf shows erythema and induration. Immediate treatment with oxygen, 4L/min via nasal cannula, is initiadet. Arterial blood gas analysis shows:

ph: 7.54
pCO2: 29 mmHg
pO2: 61 mmHg

CT angiography of the chest shows a large embolism occluding blood flow to the right lower lobe. Which of the following terms best describes the current ventilation-perfusion relationship in the right lobe of this patient?

A) Dead space
B) Diffusion abnormality
C) Hypoventilation
D) Low FiO2
E) Shunt

I put B and I got wrong. What is the right answer?
 
Two sets of patients (Group X and Y) of similar age, weight, gender, and health status are given general anesthesia by inhalation. In Group X, anesthesia is induced by administering isoflurane at a concentration 1.5 times the minimal alveolar concentration (MAC) for this drug. In Group Y, anesthesia is induced by administering a combination of isoflurane and nitrous oxide, each at 0.75 times the MAC for that drug. The onset of anesthesia is found to be significantly faster in Group Y than in Group X. The different response rates between the two groups is best explained by differences in which of the following properties of isoflurane and nitrous oxide?

A) blood:gas partition coefficient
B) brain:blood partition coefficient
C) brain:gas partition coefficient
D) hepatic metabolism
E) lipid solubility
F) minimal alveolar concentration
G) molecular wight

I put E answer and I got wrong. What is the right answer?
 
Two sets of patients (Group X and Y) of similar age, weight, gender, and health status are given general anesthesia by inhalation. In Group X, anesthesia is induced by administering isoflurane at a concentration 1.5 times the minimal alveolar concentration (MAC) for this drug. In Group Y, anesthesia is induced by administering a combination of isoflurane and nitrous oxide, each at 0.75 times the MAC for that drug. The onset of anesthesia is found to be significantly faster in Group Y than in Group X. The different response rates between the two groups is best explained by differences in which of the following properties of isoflurane and nitrous oxide?

A) blood:gas partition coefficient
B) brain:blood partition coefficient
C) brain:gas partition coefficient
D) hepatic metabolism
E) lipid solubility
F) minimal alveolar concentration
G) molecular wight

I put E answer and I got wrong. What is the right answer?

It's A. Cant explain it well though.
 
in a cough, what happens to the vocal cords while swallowing, after irritation, while coughing?
is close, close and open?
 
An otherwise healthy 60-year-old man comes to the physician for a follow-up examination 2 weeks after he was found to have a serum prostate-specific antigen concentration of 5.3 ng/mL (N<4). A biopsy specimen of the prostate shows evidence of adenocarcinoma. A CT scan of the spine shows osteobastic lesions at L1. The patient undergoes bilateral orchiectomy. Five months later, he dies suddenly of a massive myocardial infarction. At autopsy, which of the following microscopic changes in the prostate resulting from the orchiectomy is most likely present?

A) Corpora amylacea
B) Edema
C) Hypertrophied cells
D) Necrosis of the blood vessel walls
E) Scattered shrunken cells
F) Widespread necrosis

I put C and I got it wrong. What is the right answer?
 
A 32-year-old woman, gravida 4, para 0, aborta 3, delivers a female newborn at term with dismorphic features and numerous organ anomalies. The mother has had multiple spontaneous abortions, and this is her first liveborn child. Which of the following is the most likely explanation for the abnormalities seen in this newborn?

A) Autosomal dominant inheritance
B) Autosomal recessive inheritance
C) Gonadal mosaicism
D) Submicroscopic deletion
E) Unbalanced chromosome rearrangement

I put C and I got it wrong. What is the right answer?

I thought on Turner s. or something like that... what is the correct diagnosis?
 
I too put mosaicism. ArmyTiger wrote earlier that he suspects E) Unbalanced chromosome rearrangement, which in retrospect I agree with. MCC of miscarriage is trisomy 16, which could present as normal pheno in the mom with unbalanced rearrangement.
 
Maybe, above all, they want you to know that an unbalanced chromosomal rearrangement is mostly incompatible with Conception and a full-term pregnancy. I mean that, among all the options, chromosome pieces defect is the worst. What do you think about this?
 
I think since she's had 3 miscarriages and the only live birth has malformations suggests something wrong in all of mom's cells that is not affecting her, but affecting her children. Unbalanced translocation in the offspring would fit this.
 
1) A previously healthy 18-yr old man is brought to the emergency department by his mother 45 minutes after she found him crouched in his closet, saying he had to hide from the intruders "from that other place." There were no intruders in his house. He has no history of alcohol or drug use. He appears afraid of the physician and sits far away from her. During the interview, he says that he has heard to male voices in his head for the past 2 weeks. Physical examination and laboratory studies show no abnormalities. Which of the following is most likely diagnosis?

A) Bipolar
B) Brief Psychotic disorder
C) Delusional disorder
D) Schizoaffective
E) Schizophrenia

I think it is B but I get confused with brief psychotic disorder vs Delusional.

2. Question was to identify the G cells on the histology slide.
Does anyone know the answer to this?

3. A 25 y/o man comes to the physician 8 hours after the onset of severe pain of his low back that radiates down his left lef. He started a weight lifting regimen earlier in the day during which he tried to lift a bar loaded with 91kg from the ground to over his head. The pain began almost immediately after this attempt. Which of the following is the most likely cause of this patients pain?

A) A fracture of a lumbar vertebra
B) Rupture of the deltoid tendon
C) Rupture of an intervertebral disc
D) A tear in the sciatic nerve
E) A torn muscle in the low back

4. an 18 year old woman comes to the physician because of progressive fever, general malaise, and blood in her urine since she began taking oral antibiotics for a urinary tract infection 5 days ago. She also has 3-day history of a rash. Her temperature is 100.4 and pulse 75, resp are 12/min, BP 125/80. Physical exam shows petechial rash over the chest back and upper and lower extremities. Urinalysis shows WBC blood protein and eosinophils.

I think the answer is Interstitial nephritis but I wasn't sure because I thought 5 days is too short of a time period to see a reaction?

5. 54 y/o woman admitted to the hospital with acute MI. No signs of ht failure or murmur. Two days later she becomes acutely short of breath and diaphoretic. Puls 100, reap 24, bp 160/98. Crackles are heard bilaterally on both lung fields. Murmur is heard as well. Which of the following is most likely the murmur?

Is this going to be mitral regurg due to rupture of the papillary muscles.

6. 54 yo woman with terminal metastatic pancreatic cancer. Comes to the physician for a follow up examination. She asks the physician to perscribe her sufficient quantity of meds to allow her to commit suicide if the pain becomes unbearable. The physician refuses to do so but he informs the patient that he will do everything he can to manage her pain. The physician's decision in this situation is most consistent with which of the following ethical principles?

A. Autonomy
B. Beneficience
C. Dignity
D. Justice
E. Nonmalfience

7. A previously healthy 7 y/o boy to the physician by his parents bc of a tender lump in his right armpit 6 days ago. Two weeks ago his parents noticed a small red, and tender bump on his right index finger. The family has several pets including cats. Temp is 100. Physical shows 2x2 cm tender lymph node in his right axilla. Microscopic exam of a biopsy specimen of the node shows a necrotizing granulomatous adenines with stellate micro abscesses. Which of the following is the most like casual organism?

A. Bartonella Henselae
B. EBV
C. Histo Capsulatum
D. M. Tuberculosis
E. Toxoplasma

I think its A?

Thanks for the help in advance
 
disc rupture on the weightlifter. down the leg is sciatic nerve involvement, disc guts protrude on the nerve root

cancer lady - nonmalefesence

cat scratch - yea bartonella -> granulomarous lymph nodes

murmur/MI - fairly certain its rupture of the papillary, but i dont remember that question specifically. was ventricular septum rupture an answer choice or am i confusing this with something else?

interstitial nephritis - i put ATN which was wrong. this question is bull**** since drugs can cause both and more importantly i suck at kidney which the nbme should be more cognizant of. but wikipedia says eosinophiluria is either TIN, UTI, or churgg-strauss so I say TIN is it..
 
Can anyone answers this? I put A and I got wrong.
 

Attachments

  • HYDROGENS.jpg
    HYDROGENS.jpg
    88.5 KB · Views: 516
They have to be parietal cells, pale-staining cells.

What is the right answer??
 
Last edited:
16 yo girl with CF, alert, oriented, vital signs normal. Not compliant to her medication regimen x6 months. She has generalized weakness, numbness and tingling of arms and legs, difficulty walking, loss of DTR, ataxic gait.

Which is deficient:
A. folic acid
B. vit A
C. vit B6
D. vit D
E. vit E
 
1) A previously healthy 18-yr old man is brought to the emergency department by his mother 45 minutes after she found him crouched in his closet, saying he had to hide from the intruders "from that other place." There were no intruders in his house. He has no history of alcohol or drug use. He appears afraid of the physician and sits far away from her. During the interview, he says that he has heard to male voices in his head for the past 2 weeks. Physical examination and laboratory studies show no abnormalities. Which of the following is most likely diagnosis?

A) Bipolar
B) Brief Psychotic disorder
C) Delusional disorder
D) Schizoaffective
E) Schizophrenia

I think it is B but I get confused with brief psychotic disorder vs Delusional.

2. Question was to identify the G cells on the histology slide.
Does anyone know the answer to this?

3. A 25 y/o man comes to the physician 8 hours after the onset of severe pain of his low back that radiates down his left lef. He started a weight lifting regimen earlier in the day during which he tried to lift a bar loaded with 91kg from the ground to over his head. The pain began almost immediately after this attempt. Which of the following is the most likely cause of this patients pain?

A) A fracture of a lumbar vertebra
B) Rupture of the deltoid tendon
C) Rupture of an intervertebral disc
D) A tear in the sciatic nerve
E) A torn muscle in the low back

4. an 18 year old woman comes to the physician because of progressive fever, general malaise, and blood in her urine since she began taking oral antibiotics for a urinary tract infection 5 days ago. She also has 3-day history of a rash. Her temperature is 100.4 and pulse 75, resp are 12/min, BP 125/80. Physical exam shows petechial rash over the chest back and upper and lower extremities. Urinalysis shows WBC blood protein and eosinophils.

I think the answer is Interstitial nephritis but I wasn't sure because I thought 5 days is too short of a time period to see a reaction?

5. 54 y/o woman admitted to the hospital with acute MI. No signs of ht failure or murmur. Two days later she becomes acutely short of breath and diaphoretic. Puls 100, reap 24, bp 160/98. Crackles are heard bilaterally on both lung fields. Murmur is heard as well. Which of the following is most likely the murmur?

Is this going to be mitral regurg due to rupture of the papillary muscles.

6. 54 yo woman with terminal metastatic pancreatic cancer. Comes to the physician for a follow up examination. She asks the physician to perscribe her sufficient quantity of meds to allow her to commit suicide if the pain becomes unbearable. The physician refuses to do so but he informs the patient that he will do everything he can to manage her pain. The physician's decision in this situation is most consistent with which of the following ethical principles?

A. Autonomy
B. Beneficience
C. Dignity
D. Justice
E. Nonmalfience

7. A previously healthy 7 y/o boy to the physician by his parents bc of a tender lump in his right armpit 6 days ago. Two weeks ago his parents noticed a small red, and tender bump on his right index finger. The family has several pets including cats. Temp is 100. Physical shows 2x2 cm tender lymph node in his right axilla. Microscopic exam of a biopsy specimen of the node shows a necrotizing granulomatous adenines with stellate micro abscesses. Which of the following is the most like casual organism?

A. Bartonella Henselae
B. EBV
C. Histo Capsulatum
D. M. Tuberculosis
E. Toxoplasma

I think its A?

Thanks for the help in advance

Just the stuff that other people haven't said yet:

4. Pathoma makes a big deal that eosinophils in urine = Interstitial nephritis. I think you should think that until proven otherwise.

5. Yep, Mitral Regurg. Doesn't have to be rupture (since the time period was too soon for rupture in the Q stem), it could just be hypokinetic.

7. Bartonella henselae. Stellate granulomas are actually pathognomonic (from Pathoma as well, actually!)
 
Last edited:
16 yo girl with CF, alert, oriented, vital signs normal. Not compliant to her medication regimen x6 months. She has generalized weakness, numbness and tingling of arms and legs, difficulty walking, loss of DTR, ataxic gait.

Which is deficient:
A. folic acid
B. vit A
C. vit B6
D. vit D
E. vit E

E. Vitamin E.

Cystic fibrosis --> fat soluble vitamin deficiency. Answer choices B, D, E are left: only one that causes neuro symptoms is Vitamin E.
 
A 43-year-old woman comes to the physician because of progressive difficulty walking during the past 3 months. Neurologic examination shows weakness and decreased muscle bulk of the lower extremities. Patella and Achilles tendon reflexes are diminished. Sensations of join position, pain, and temperature in the lower extremities are normal. Which of the following is the most likely cause of the findings in this patient?

A) Acute peripheral neuropathy
B) Degeneration of motoneurons of the lumbar cord
C) Demyelination of the corticospinal pathways
D) Loss of afferent la axons innervating muscle spindles
E) Myotonic muscolar dystrophy

I put C and it's wrong, What is the right answer?
 
Can anyone help with any of these?

1. The 8 year old girl w/ breast buds + pubic hair
I know the answer is "normal development", but how the hell is 8 years old a normal pubertal age? Every resource I look at says that her Tanner stage is >10 yrs, and this site says "puberty is generally considered to be early if it starts around age 7 or 8": http://www.dukemedicine.org/blog/when-puberty-too-early

2. Catheters are measuring lymph flow in the lungs, and you can mess with variables to see how they effect lymph flow. You can inject anything IV or change the ventilated air and see how it effects lymph flow. Which increases lymph flow in the lungs?
Why was the answer to this one 0.9% NaCl infusion? Couldn't phenylephrine infusion also increase hydrostatic pressure?

3. Woman has had 3 spontaneous abortions, and her at-term newborn has dysmorphic features + numerous organ anomalies. Why?
I know the right answer is unbalanced translocation, but I have two questions about this:
-Where would the mother's unbalanced translocation have occurred? It would have had to have been in very early development to affect all her cells right?
-Why is gonadal mosaicism incorrect?

4. Guy from Puerto Rico can't speak English, and is getting a colonoscopy to see if he has cancer. His family doesn't want him to know the outcome. Reply?
I put that HIPAA requires that the patient receive his own information and make his own decisions, but that was wrong. Do you get an interpreter to ask him how much he wants to know? Was the HIPAA answer wrong because HIPAA allows patients to waive the right for informed consent?

Couldn't believe that pseudofolliculitis barbae question. I had it down to that or rosacea, but whoops, neither of them are in FA, Pathoma, or UWorld so I didn't know.
 
Last edited:
A 43-year-old woman comes to the physician because of progressive difficulty walking during the past 3 months. Neurologic examination shows weakness and decreased muscle bulk of the lower extremities. Patella and Achilles tendon reflexes are diminished. Sensations of join position, pain, and temperature in the lower extremities are normal. Which of the following is the most likely cause of the findings in this patient?

A) Acute peripheral neuropathy - Incorrect; This would show sensory signs.
B) Degeneration of motoneurons of the lumbar cord
C) Demyelination of the corticospinal pathways -- Incorrect; This would UMN signs such as increased reflexes.
D) Loss of afferent la axons innervating muscle spindles -- Incorrect; I'm pretty sure this wouldn't cause weakness + decreased bulk of muscles
E) Myotonic muscolar dystrophy -- Incorrect; No signs of myotonia

I put C and it's wrong, What is the right answer?

Answer in quote.
 
Can anyone help with any of these?

1. The 8 year old girl w/ breast buds + pubic hair
I know the answer is "normal development", but how the hell is 8 years old a normal pubertal age? Every resource I look at says that her Tanner stage is >10 yrs, and this site says "puberty is generally considered to be early if it starts around age 7 or 8": http://www.dukemedicine.org/blog/when-puberty-too-early

i don't know but the right answer is that

2. Catheters are measuring lymph flow in the lungs, and you can mess with variables to see how they effect lymph flow. You can inject anything IV or change the ventilated air and see how it effects lymph flow. Which increases lymph flow in the lungs?
Why was the answer to this one 0.9% NaCl infusion? Couldn't phenylephrine infusion also increase hydrostatic pressure?

Lymph flow is mainly due to venous return, so Water0.9%NaCl infusion gets this


3. Woman has had 3 spontaneous abortions, and her at-term newborn has dysmorphic features + numerous organ anomalies. Why?
I know the right answer is unbalanced translocation, but I have two questions about this:
-Where would the mother's unbalanced translocation have occurred? It would have had to have been in very early development to affect all her cells right?
Yes, I think so
-Why is gonadal mosaicism incorrect?
Gonadal mosaicism havs better hopes to give a full term baby than unbalanced translocation

4. Guy from Puerto Rico can't speak English, and is getting a colonoscopy to see if he has cancer. His family doesn't want him to know the outcome. Reply?
I put that HIPAA requires that the patient receive his own information and make his own decisions, but that was wrong. Do you get an interpreter to ask him how much he wants to know? Was the HIPAA answer wrong because HIPAA allows patients to waive the right for informed consent?

interpreter is often the best answer in this kind of questions, because informed consense must be made only if the patient can really understand what you are speaking about

5. Guy with horseshoe kidney

Couldn't believe that pseudofolliculitis barbae question. I had it down to that or rosacea, but whoops, neither of them are in FA, Pathoma, or UWorld so I didn't know.

i don't remember the question, but maybe it was simply tinea barbis

answers in green in quote
 
  • Like
Reactions: 1 user
lymph flow Q - phenyl will cause constriction before the capillary bed and reduce filtration. IVF will increase pressure at the capillary bed and increase lymph

unbalanced tranny - yes the mother has 100% of her cells looking like
X Z
X Z
X
Z
Z
Z

instead of:

X Z
X Z
X Z
... Z
... Z


so any chromosome she gives off will be damaged. probably the abortions all got the ZZ chromosome and the liveborn got the XXXZZ but either way its incompatable with normal dev.

Puerto rican - yeah its get a translator. the answer will almost NEVER be "do such and such because of a law/administrative guideline." as physicians we only answer to the utmost authority, which is the principles of ethics which can be distilled from the aether by pure logic.

and conan that Q about motor neurons is not a diagnosis questino - its a "do you understand what an isolated LMN defect is Q"
 
Last edited:
  • Like
Reactions: 2 users
Thanks to both of you!

What is the diagnosis? Can you explain why u choosed this answer? Thanks

What witzelsucht said.

lymph flow Q - phenyl will cause constriction before the capillary bed and reduce filtration. IVF will increase pressure at the capillary bed and increase lymph

unbalanced tranny - yes the mother has 100% of her cells looking like
X Z
X Z
X
Z
Z
Z

instead of:

X Z
X Z
X Z
..... Z
.... Z


so any chromosome she gives off will be damaged. probably the abortions all got the ZZ chromosome and the liveborn got the XXXZZ but either way its incompatable with normal dev.

Puerto rican - yeah its get a translator. the answer will almost NEVER be "do such and such because of a law/administrative guideline." as physicians we only answer to the utmost authority, which is the principles of ethics which can be distilled from the aether by pure logic.

and conan that Q about motor neurons is not a diagnosis questino - its a "do you understand what an isolated LMN defect is Q"

Great stuff. So is an "unbalanced tranny" an ataxic cross-dresser?

I'm assuming if she were a gonadal mosaic (say she were: 45, XX -16 / 46, XX), then it would be impossible for her to have anything but a stillbirth because every oocyte would be 45, XX, -16?

Anyone have any further thoughts on the 8 year old pubescent girl?
 
  • Like
Reactions: 2 users
Thanks to both of you!



What witzelsucht said.



Great stuff. So is an "unbalanced tranny" an ataxic cross-dresser?

I'm assuming if she were a gonadal mosaic (say she were: 45, XX -16 / 46, XX), then it would be impossible for her to have anything but a stillbirth because every oocyte would be 45, XX, -16?

Anyone have any further thoughts on the 8 year old pubescent girl?

The answer for the 8 year old pubescent is normal development. Seems wrong, but kids are starting puberty earlier and earlier, and clinically you should also consider when her mother and sisters started puberty, might just be normal for them.
Also, seems like every development question I get, the answer is normal development.
 
The answer for the 8 year old pubescent is normal development. Seems wrong, but kids are starting puberty earlier and earlier, and clinically you should also consider when her mother and sisters started puberty, might just be normal for them.
Also, seems like every development question I get, the answer is normal development.

Wow, I had no idea. Thanks!
 
A 2-year-old boy is brought to the physician because of developmental delay. There is a history of hearing loss in his mother and delayed speech in his older sister. His maternal uncle had stroke-like episodes at the age of 25 years. Physical examination shows ophthalmoplegia and hypotonia. His serum lactic acid concentration is increased. Which of the following best explains the findings in this family?

A) Autosomal dominant inheritance
B) Decreased penetrance
C) Heteroplasmy
D) Imprinting
E) X-linked recessive inheritance

What is the correct answer?

What is the diagnosis? So many diseases in the relatives... i have no idea :(
 
Top